İspat et $\lim_{n\rightarrow\infty}\frac{n\log(n)}{\log(n!)} = 1$[kopyalamak]

Jan 25 2021

kanıtlamam gerek$$\lim_{n\rightarrow\infty}\frac{n\log(n)}{\log(n!)} = 1$$ama aklımın ucundayım. Web'i taradım ama yalnızca ters = 0'ı kanıtlayan örnekler/cevaplar bulabiliyorum ve kendim için boşuna olmayan bir sürü yöntem denedim (genişletilmiş terimler, hepsi yanlış olan sayısız farklı türevlerle L'Hopitals Kuralı). Biri beni doğru yöne işaret edebilir mi, tamamen sıkışıp kaldım ...

Yanıtlar

3 BenjaminWang Jan 25 2021 at 08:52

Bunu not et$\log n! = \sum_{k=1}^n \log k$. İlgili grafikleri çizerek şunları görebilirsiniz:

$$\int_1^n \log x dx \le \sum_{k=1}^n \log k $$

$$\le \int_1^{n+1} \log x dx$$

Şimdi integrali hesaplayın$\int_1^m \log x dx = m \log m - m + 1$, böylece yukarıdaki olur

$$n \log n - n + 1 \le \log n! \le (n+1)\log(n+1)-n$$

Ve şimdi, bölme işleminden sonra sıkıştırma teoremi ile sonucunuzu alıyoruz.

1 crystal_math Jan 25 2021 at 09:04

$\log(n!)\ge \frac{n}{2}\log(\frac{n}{2})$ve bu yüzden$\dfrac{n\log(n)}{\log(n!)}\le \dfrac{n\log(n)}{\frac{n}{2}\log(\frac{n}{2})}$

Üst sınırın bu sınırını değerlendirirken,$2$o zamandan beri$\lim_{n\rightarrow \infty} \dfrac{\log(n)}{\log(n/2)} = 1$. Ancak, seçerseniz$\epsilon >1$, Anlıyorsun

$\log(n!)\ge \frac{n}{\epsilon}\log(\frac{n}{\epsilon})$ve bu yüzden$$\dfrac{n\log(n)}{\log(n!)}\le \dfrac{n\log(n)}{\frac{n}{\epsilon}\log(\frac{n}{\epsilon})}\rightarrow \epsilon$$

dan beri$\epsilon>1$(keyfi), şu sonuca varabilirsiniz:$$\dfrac{n\log(n)}{\log(n!)}\le 1$$

(alt sınırı kolayca elde edebilirsiniz) ve bu nedenle sınır$1$.

1 zkutch Jan 25 2021 at 08:58

kullanma$$\left( \frac{n}{e}\right)^n \lt n! \lt e \left( \frac{n}{2}\right)^n$$sahibiz$$n \log \frac{n}{e} \lt \log n! \lt \log e+ n \log \frac{n}{2}$$

Ek.

Sol taraf için tümevarımın ilk adımı açıktır. O zamanlar$$(n+1)!=n!(n+1) \gt \left( \frac{n}{e}\right)^n (n+1) = \\ =\left( \frac{n+1}{e}\right)^{n+1} \frac{(n+1)\left( \frac{n}{e}\right)^n}{\left( \frac{n+1}{e}\right)^{n+1}} \gt \left( \frac{n+1}{e}\right)^{n+1}$$Çünkü$(n+1)\left( \frac{n}{e}\right)^n \left( \frac{n+1}{e}\right)^{-n-1}\gt 1$eşdeğerdir$\left(1+ \frac{1}{n}\right)^{n} \lt e$.

Sağ taraf için$$n! \lt \left(\frac{n+1}{2}\right)^{n} = e\left(\frac{n}{2}\right)^{n} \frac{\left(\frac{n+1}{2}\right)^{n}}{e\left(\frac{n}{2}\right)^{n}} = \\ =e\left(\frac{n}{2}\right)^{n} \frac{\left(1+ \frac{1}{n}\right)^{n}}{e} \lt e\left(\frac{n}{2}\right)^{n}$$

UNOwen Jan 25 2021 at 09:00

$$\displaystyle \frac{x\ln \left(x\right)}{\ln \left(x!\right)}=\frac{x\ln\left(x\right)}{\ln\left(\Gamma \left(x+1\right)\right)}$$

L'Hôpital kuralının uygulanması,

$$\lim _{x\to \infty }\left(\frac{x\ln \left(x\right)}{\ln \left(\Gamma \:\left(x+1\right)\right)}\right)=\lim_{x\to \:\infty \:}\left(\displaystyle \frac{\ln(x)+1}{\psi \:^{\left(0\right)}\left(x+1\right)}\right)$$

Tekrar uygulama, verim

$$\lim _{x\to \infty }\left(\frac{\frac{1}{x}}{\psi ^{\left(1\right)}\left(x+1\right)}\right)=\lim _{x\to \infty }\left(\frac{1}{x\left(\psi ^{\left(1\right)}\left(x+1\right)\right)}\right)$$

Payda 1'e şu şekilde yaklaşır:$x\rightarrow \infty$.